Important Announcement
PubHTML5 Scheduled Server Maintenance on (GMT) Sunday, June 26th, 2:00 am - 8:00 am.
PubHTML5 site will be inoperative during the times indicated!

Home Explore NY AND RELATED AGGREGATES CASE STUDY

NY AND RELATED AGGREGATES CASE STUDY

Published by Praneet Mohan, 2021-01-02 14:28:50

Description: NY AND RELATED AGGREGATES CASE STUDY

Search

Read the Text Version

NATIONAL INCOME AND RELATED AGGREGATES CASE STUDY Q.1 Net Factor Income from Abroad refers to the difference between factor income received from the rest of the world and factor income paid to the rest of the world. NFIA is significant to differentiate between Domestic Income and National Income. In Practical estimates, domestic income is estimated first and then, National Income is derived from Domestic Income. 1. How is National Income derived from Domestic Income? 2. What is meant by Factor income from abroad? 3. NFIA is _________ when income earned from abroad is less than income paid to abroad. 4. What is the value of NFIA in a closed economy? ANSWERS: 1. Domestic Income + NFIA = National Income 2. The income earned by the normal residents of the country from the rest of the world in the form of wages and salaries, rent, interest, dividend and retained earnings. 3. Negative 4. Zero Q.2 Case study based (National income) Read the following news report and answer Ques 1- 4 ( each carries 1 mark) on the basis of the same: The Reserve Bank of India (RBI) in its latest monthly bulletin (November) has dedicated a chapter on the 'State of the economy' to highlight the key indicators of India‘s economic health. The RBI has now started 'nowcasting' or in simpler terms 'the prediction of the present or the very near future of the state of the economy'. The first 'nowcast' has predicted that the country's economy will shrink by 8.6% in the second quarter (July, August, September) of the current Fiscal Year. This means that India has entered a 'technical recession' in the first half of the current FY for the first time in history. It is important to note that the GDP had shrunk by 23.9% in the first quarter (April, May, June). India's GDP growth contracts by 23.9%: The RBI Bulletin; March 27th, 2020 1. The continuous fall of GDP leads to ……………………………(choose the correct alternative) (a) Rise in national income (b) Fall in national income (c) No change in national income (d) fall in general price level

2. In the situation of continuous fall in demand level in economy it is likely to happen that a) Amount of subsidies and other transfer payments in national income rises b) Amount of subsidies and other transfer payments in national income falls c) Amount of taxes is likely to increase in order to generate additional income for govt. d) None of the above 3. Under the scheme of ‗Aatamnirbharbharat‘ govt. has given a huge amount of loan to different sectors of economy particularly to promote MSME. The interest paid by MSME also has loan moratorium period. Will these interest payments by MSMEs be included in national income? Give reason. 4. Due to this ‗Covid 19‘ pandemic a lots of Indians sent back to India. As a result these NRIs have to come back and have been staying in India and working from home. Identify which statement regarding their status in India is true a) They are treated as normal residents of India b) Their income will be treated as factor income from abroad c) Their income will be treated as factor income to abroad d) No effect on National income of India. Q.3 Read the following statement and answer Questions 7-10 on the basis of the same: One‘s ability to buy commodities comes from the income one earns as labourer (earning wages), or as entrepreneur (earning profits), or as landlord (earning rents), or as owner of capital (earning interests). In short, the incomes that people earn as owners of factors of production are used by them to meet their demand for goods and services. Q7. The statement above is based on the notion‖ one‘s expenditure is other man‘s income. (true /false) Q8. This exchange is depicted as ----------(real flow/money flow) Q9. Using up of incomes earned in consumption of goods is called …….( income distribution/income disposal) Q10. All the factors of production are owned by ------ household/firms) Ans: 7. True 8. money flow 9. income disposal 10. households Q.4 New Delhi: Asserting that the ― worst is behind us‖ veteran banker Deepak Parekh on Thursday said that India‘s GDP growth will be in positive territory in the fourth quarter as demand has started picking up in all sectors . citing an example he said during the virtual conference organized by Canada – India- based council. He said toll collections are back to

88 percent of pre COVID 19 level , e way bills are increasing and electricity consumption has stabilized. India is domestic based economy and demand is set to rise in coming future. Source : 9 october 2020, The Economic Times On the basis of above case let answer the following questions (4 M) A. Do you think higher level of Real GDP always lead to higher availability of goods per person in the domestic economy? Comment B. There is no difference between GDP at market price and GDP at factor cost in two sector economy. Comment C. Can we say that GDP is true indicator of welfare? Justify D. Real GDP is considered as an index of_________________. Q.5 Case Study GDP and welfare Can the GDP of a country be taken as an index of the welfare of the people of that country? If a person has more income, he or she can buy more goods and services and his /her material being improves. So it may seem reasonable to treat his or her income level. GDP is the sum total of value of goods and services created within the geographical boundary of a country in a particular year. It gets distributed among the people as incomes (except for retained earnings). So we may be tempted to treat higher level of GDP of a country as an index of greater well being of the people of that country(to account for price changes, we may take the value of real GDP instead of nominal GDP) Source: Introductory Macoeconomics Textbook for class 12, p.no 27 Questions based on case study 1. Is it reasonable to treat a person‘s income level as his or her level of well-being? Give reason 2. Which is the better measurement of GDP? (Nominal GDP/Real GDP) Why? 3. Do you think that GDP is a true indicator of welfare?(Yes/No) 4. In the given picture, do you think that the distribution of GDP is uniform?

Answers: 1. No. A person‘s income alone will not determine his well being. Other determinants like health, happiness, etc also influence the well being of a person. 2. Real GDP is the best measure of National income as it considers the constant price to compare physical output of goods and services over different years. 3. GDP is often considered as an index of welfare. Higher GDP is generally taken as greater welfare of people. But GDP alone does not determine the welfare as it has following limitations a) Inequalities of distribution b) Non-monetary exchanges are not included in GDP c) Negative externalities like environmental pollution reduces the welfare on health d) GDP does not consider the changes in the population of a country 4. No. There is inequality between rich and poor. Q.6 The growth our tourists were talking about was in Gross Domestic Product (GDP), which measures the final value of all goods and services produced by an economy. But GDP measures quantity not quality. In other words, although it says a great deal about how much stuff you can churn out, it tells you little about state of your economic development. E.g. GDP counts all investment as positive, whether or not that investment turns out to be productive in the longer run. GDP fails to account for the cost of environmental damage. All production is regarded as positive even if the pollution it causes reduces the production capacity or pushes up health care costs. (SOURCE: South China Post May 25,2011) Q1 Define Gross Domestic Product at Market Price. A1 It refers to gross market value of all final goods and services produced within the domestic territory of a country during a period of one year. Q2 Why comparing the GDP of various nations might not tell you which nation is better off? A2 The well being of nation or standard of living is measured by per capita income i.e. GDP/Total Population and not only by GDP. Q3 What is Green GNP? A3 Green GNP measures national income or output adjusted for the depletion of natural resources and degradation of the environment. Q4 GDP calculation does not directly include the social costs of environmental damages like global warming, acid rain etc. Do you think these costs should be included in GDP. Why or why not? A4 Yes because people‘s well being is affected by these environmental damages.

Q.7 Case Based Question On November 12,2020,Finance Minister Smt. NirmalaSitharaman announced the fourth economic stimulus package. During the announcement the Finance Minister introduced the AtmaNirbhar Bharat Rojgaryojna‖.The measures included the production linked incentive scheme for 10 sectors with a purposed expenditure of 1.46 lakh crore over five years. Source : The Hindu 13 November 2020 Que. 1 What is the need of Stimulus package? Que2. What is production incentive linked scheme? Que 3. What is the purpose of AtmaNirbhar Bharat Rojgaryojna? Que 4. What is the need of PLI scheme? Ans. 1 .To boost job creation, To provide liquidity support to stressed sectors Ans 2. PLI scheme aims to give companies incentives on incremental sales from manufactured in domestic units. Ans 3. To Spur Job creation Ans 4.To attract investment, To enhance exports Read the following statements- Assertion (A) and Reason (R). Choose one of the correct alternatives given below: Q.8 CASE STUDY: India is forming a national database of natural resource accounting (NRA) that is to help calculate the cost of recovery of degraded resources, as well as to calculate green GDP. It is expected that the value of green GDP will be significantly lower than conventionally

measured real GDP. China had similarly calculated its green GDP in 2004 and had found losses due to environmental damage valued at 3% of GDP. Economists have debated the concepts of NRA and green GDP over many years, but with little agreement over how natural resources should be valued. These concepts are now gaining acceptance because of growing recognition of the limitations of GDP as the traditional indicator of growth. Green GDP will take into account of the current situation of resource depletion and will provide a long-term perspective on economic growth and its effects. Source: Adapted from SuchmiDey and Anto Antony, ‗Green resources to colour GDP‘ in The Economic Times, 11 June 2009. (i) What is Green GDP? (ii) Which one is better? Real GDP or Nominal GDP. (iii) Why is Green GDP expected to be lower than conventionally measured GDP? (iv) What might be the benefits for India from calculations of green GDP? (any one) SOLUTIONS: (i) Green GDP measures domestic income or output adjusted for the depletions of natural resources and degradation of the environment. (ii) Real GDP as it truly reflects the growth of an Economy. (iii) It is lower because it is involves the subtraction of GDP from all the costs of environmental externalities and pollution . (iv) It will help to attain a sustainable use of natural environment and equitable distribution of benefits of developments. Q.9 Case Study Gross Domestic Product (GDP) defines the economic worth of products and services manufactured in a country in a definite financial year. It also accounts for the revenue received by foreign citizens locally and the insufficient income earned by country‘s residents abroad. Whenever the measurement of GDP is at current values, it represents nominal GDP and the real GDP is evaluated at fixed prices. Both the GDP are financial tools for estimating a nation‘s economic development and growth. However, there is still confusion on which GDP indicates the nation‘s development better. Q1.GDP Inflation is adjusted in Nominal GDP .( True/ False) 1 Q2. When can Real GDP be greater than Nominal GDP? 1 Q3. If Real GDP is ₹ 500 and price index is 125, calculate the Nominal GDP. 1 Q4. The ratio of Nominal GDP to Real GDP is a well-known index of prices, called_________. 1 Q5. Distinguish between Real GDP and Nominal GDP 3 Answers: 1. False 2. When base year prices are higher than the current year prices. 3. ₹ 625 4 GDP Deflator

Q.10 Case based questions Read the following paragraph carefully and answer the questions on the basis of the same:— The overall objective of political activities in Democratic societies is to ensure the highest possible level of welfare for the countries citizens now and in the future. When politicians ,macroeconomists and others discuss welfare and economic development ,they often focus on Gross domestic product(GDP). GDP is a measure of the economic prosperity of a country compiled as output or income.However,GDP is regularly criticised for not presenting a fair view of welfare.If GDP is a poor measure of welfare, focusing one-sidedly on increasing GDP may lead to misguided political decisions. For example Nobel laureate Joseph Stiglitz has argued that the objective of the highest possible GDP growth will result in reduced welfare. The criticism of GDP is not new.and over last 30-40 years ,efforts have been made to put together alternative objectives.The debate has revived recently, partly as a result of publication of recent report . Q 1 _________ (RealGDP /NominalGDP) is considered as a better indicator of economic growth of an economy. Q 2 GDP is not considered as a better measure of development.Which of the following reasons could be accounted for same? i.Externalities. ii. Non marketable services iii.Production of harmful products iv.All of these Q 3 A course of action by one economic agent which has an adverse impact on other economic agent without cost sharing is called ___________ externality. Q 4 Which of the following measures of GDP considers base years‘ price for calculations of gross income? i. Nominal GDP ii. Green GDP iii. GDP deflator iv. Real GDP Answers:——— 1.Real GDP 2.All of these 3.Negative 4.Real GDP Q.11 Case based questions Read the following case study paragraph carefully and answer the questions on the basis of the same. The overall objective of political activities in democratic societies is to ensure the highest possible level of welfare for the country's citizens now and in the future. When politicians, macroeconomists and others discuss welfare and economic development, they often focus on the Gross Domestic Product (GDP). GDP is a measure of the economic prosperity of a country compiled as output or income. However, GDP is regularly criticised for not presenting a fair view of welfare. If GDP is a poor measure of welfare, focusing one-sidedly on increasing GDP may lead to misguided political decisions. For example, Nobel laureate Joseph Stiglitz has argued that the objective of the highest possible GDP growth will result in reduced welfare. The criticism of GDP is not new, and over the last 30-40 years, efforts have been made to put together alternative objectives. The debate has revived recently, partly as a result of the publication of a recent report concerning the limitations of GDP as a measure of economic

performance and social progress. The report was written by the two Nobel laureates for economy, Joseph Stiglitz and AmartyaSen, among others. 1______ (Real GDP/Nominal GDP) is considered as a better measure of economic growth of an economy. Ans Real GDP 2. GDP is not considered as a better measure of development. Which of the following reasons could be accounted for the same? (a) Externalities (b) Non-marketable services (c) Production of harmful goods (d) All of these Ans (d)All 3. A course of action by one economic agent which has an adverse impact on other economic agent without cost sharing, is known as _______(positive/negative) externality. Ans Negative 4. Which of the following measures of GDP considers base year's price for calculation of gross income? (a) Green GDP (b) Nominal GDP (c) GDP deflator (d) Real GDP Ans (d) Real GDP Q.12 CASE STUDY Can the GDP of a country be taken as an index of the welfare of the people of that country? If a person has more income, he or she can buy more goods and services and his or her material well- being improves. So, it may seem reasonable to treat his or her income level as his or her level of well-being. GDP is the sum total of value of goods and services created within the geographical boundary of a country in a particular year. It gets distributed among the people as incomes (except for retained earnings). So, we may be tempted to treat higher level of GDP of a country as an index of greater well-being of the people of that country. Answer the following question- 1. Is it reasonable to treat a person's income level as his or her level of wellbeing? 2. What is GDP? 3.Do you think that the distribution of GDP is uniform? 4. Do you think that GDP is a true indicator of welfare? Q.13 National Income Refer the following figure and answer Questions 1-4 on the basis of the same: Gross Domestic Product measures the aggregate production of final goods and services taking place within the domestic economy during a year. But the whole of it may not accrue to the citizens of the country. For example, a citizen of India working in Saudi Arabia may be earning her wage and it will be included in the Saudi Arabian GDP. But legally speaking, she is an Indian. Is there a way to take into account the earnings made by Indians abroad or by the factors of production owned by Indians? When we try to do this, in order to maintain symmetry, we must deduct the earnings of the foreigners who are working within our domestic economy, or the payments to the factors of production owned by the foreigners. For example, the profits earned by the Korean-owned Hyundai car factory will have to be subtracted from the GDP of India. The macroeconomic variable which takes into account such additions and subtractions is known as Gross National Product (GNP).

1. Gross Domestic Product measures the aggregate ________ of final goods and 1 services taking place within the domestic economy during a year. Production 1 2. State the concept in the passage related to ‗additions and subtractions to GDP‘. Net factor income from abroad 3. Discuss an example given in the passage related to the figure. 1 The profits earned by the Korean-owned Hyundai car factory will have to be subtracted from the GDP of India. 4. GNP takes macroeconomic variable related to payments and receipts into account by: 1 a) Subtractions b) Additions c) Additions and subtractions d) Multiplication and Division Q.14 The economics of firecrackers As the air quality worsened in northern parts of India and the Covid-19 pandemic witnessed a new wave in the national capital in the festive season, all focus shifted to firecrackers and their sales across the country. India‘s firecracker industry is estimated to be around Rs 5,000 crore, with the majority of the production in Sivakasi in Tamil Nadu. However, government flip-flops and intervention by courts have plunged it into a crisis. The uncertainty around sales of green crackers allowed by the Supreme Court in 2018 has not helped the industry either. In a statement on 8 November, B.C. Bhartia, president of the Confederation of All India Traders, pointed out that the firecracker industry provides employment to about 10 lakh people and estimated the total manufacturing at Rs 5,000 crore. ―In the absence of any clear policy on firecrackers, the industry is dying its own death,‖ he said. The Tamil Nadu Fireworks and Amorces Manufacturers Association (TNFAMA) estimates that there are 1,000 licensed cracker units in Sivakasi with a market size of Rs 2,500 to Rs 3,000 crore. But there are several units that are unregistered and operate out of sheds and small homes. ―On an average, production has dropped by 30 per cent,‖ said TNFAMA president P. Ganesan. He added that many of the manufacturers take huge loans during the festival season. If they are unable to sell the crackers, they will face heavy financial losses, further impacting Sivakasi‘s economy and people depending on it. A lot of these manufacturers are producing green crackers in Sivakasi, though many are unable to get the formula to make green crackers the right way. Source The Financial Express, 8th November 2020 Q1. What will be the effect of ban on sale of firecrackers on India‘s GDP? a. GDP will increase b. GDP will decrease c. GDP will not be affected d. GDP may increase or decrease. Ans. b) GDP will decrease Q2. Policy makers argue that burning of fire crackers lead to decrease in social welfare in the economy. This represents an example of : a. Positive externality b. Negative externality c. Increase in GDP d. Non-marketable activities

Ans. b) Negative externality Q3. ―There are several units that are unregistered and operate out of sheds and small homes.‖ The incomes of such units will be taken under the category of a. Operating surplus b. Income from property c. Income from entrepreneurship d. Mixed income of self employed Ans. d) Mixed income of self employed Q4. ―The firecracker industry provides employment to about 10 lakh people and estimated the total manufacturing turnover at Rs 5,000 crore.‖What is the other name for turnover used in national income accounting? a. Value added b. Mixed income c. Value of output d. Capital formation Ans. Value of output Q.15 Consumption is the final aim of all economic activities. But it must be preceded by production, distribution and exchange. National income is the final outcome of all economic activities of a nation. In India, a systematic measurement of National income was first attempted in 1949. Now the task of estimating national income is performed by CSO in its publication Estimates of National income. National income may be considered of a closed economy or an open economy. National income is the aggregate money value of all goods and services produced in a country during one year. The factor income is produced by factors of production and hence distributed between them. However, it should be noted that what is produced is more important. Q1. Which of the following are economic activities? i) Distribution ii)Production iii) Consumption iv) All of the above Q2.What is the full form of CSO i) Central service organisation ii) Central Statistics office iii) Central statistical organisation iv) Country service organisation Q3. What is the need for the study of national income? i) To measure the size of the economy

ii) To measure the level of economic performance of the country iii) To make international comparison of peoples living standard iv) To help government to formulate plans and policies to increase growth rate a) i,ii,and iv b) ii,iii,and iv c) i,iii and iv d) All of the above Q4. In a closed economy _________ is not included. i) Household ii) Firms iii) Government iv) Foreign sector Q5. In the estimation of national income, only the value of ___________goods is taken into account.( Intermediate/Final) Q6. ___________ is the year during which production of goods and services is estimated in the domestic economy.(Accounting year/Calendar year) Q7. Which of the following is not included in factor income? i) Rent ii) Wages iii)Gifts from abroad iv) Profit Q8. Higher production of _________ goods leads to higher level of welfare of the economy. (Consumption/Capital) Q9. National income does not include data from the following activities i) Income from illegal activities ii) Work done by housewives iii) Income from lotteries iv) Production of goods for self-consumption

a) i,iii and iv b) ii,iiiand iv c) i,ii and iii d) iv,i and ii Q.16 Read the following case study and answer the following: If growth rate of population is higher than the growth rate of GDP, then per capita GDP would become lower. In other words, per capita availability of goods and services declines. In such case also mere rise in GDP cannot become an index of economic welfare. Q-1 What is GDP? Q-2 What do you mean by per capita availability decline ? Q-3 Which limitation of economic welfare , discussed in above case ? Ans 1 Total production of goods and services produced in an economy over a period of time in all sectors of an economy. Ans 2 As growth rate of population is increasing than the growth rate of GDP so goods available to per person is decreasing. Ans 3 Growth rate of Population Q.17 Case Study Read the following extract and answer the questions given below On 21st June, 2015, the 1st World Yoga Day was celebrated. In India, it was significantly observed in Delhi, with more than 3500 people, including dignitaries from 84 nations and the prime minister NarendraModi. It was organised by the Ministry of AYUSH. It widely publicised through campaigns on various media to spread awareness about the benefits of Yoga among the masses. T-shirts with the yoga day logo, yoga mats and other related items were distributed to the enthusiasts. Trained yoga experts were appointed to help beginners who participated in the event. Q1. Government incurs expenditure to popularise yoga among the masses. What is its impact on Welfare? a. Welfare rises b. Welfare falls c. Government expenditure has no impact on welfare d. None of the above Q2. Government incurs expenditure to popularise yoga among the masses. What is its impact on GDP? a. GDP rises, because government incurs expenditure b. GDP falls, because government incurs expenditure c. No effect on GDP d. None of the above Q3. The term `welfare` refers to

a. Sense of pride in the nation b. Increase in purchase of services c. Sense of well-being among the people d. Strict control by the government Q4. GDP at constant prices is also called _________________ (per capita GDP / real GDP) Answers 1. a 2. a 3. c 4. real GDP Q.18 Case Study A US based software company set up its business in India and taken a two storeyed office building on rent @ Rs 4 lakhs per month. It invested Rs 10,00,000 for the purchase of computers for its employees. Indian government provided subsidies worth Rs 10 LAKHS for balanced regional growth. The company is incurring Rs 20 Lakhs per month on account of salary . It gave bonus worth Rs 40 Lakhs to its employees on Diwali. Interest worth Rs 5 lakhs is the mandatory payment. It earned a profit of Rs 70 Lakhs . Assuming that there is only one firm in the country, find out the national income of the country. Solution: NNPFC= Compensation to employees (salary + Bonus) + Rent+ Interest+ NFIA =280 +48 lakhs +5+(-70) =263 LAKHS WORKING NOTE Rent = 48 lakhs(4*12) Compensation to employees (salary+Bonus) = 280 lakhs[(20*12)+40] Interest = 5LAKHS NFIA(FIFA-FITA) = -70LAKHS Q.19 CASE STUDY In a simplified economy, it is assumed that there are no savings, taxes, exports and imports. During the production process, four factors of production receive income in the form of rent, wages interest and profit for their factor service. The sum total of these incomes is the factor income received during a year. The entire factor income is spent by the factors on purchase of final goods and services. The sum of these purchases is the aggregate final expenditure during the year. On the basis of the above case, answer the following questions: 1. What is the impact of the above situation on aggregate final consumption and aggregate factor payments?

2. What are final goods? (Ans: 1. Aggregate final consumption will be equal to aggregate factor payments. 2. They have crossed production boundary.) Q.20 Due to the pandemic of covid 19 the government has adopted to support people and provided free services . Giving reason explain how should the following be treated while estimating national income (1) Expenditure on free services provided by government. Ans - (1) Expenditure on free services provided by government should be included in the estimation of National Income, as it is a final expenditure of the government. (2) Payment of interest by a government firm. Ans- Payment of interest by a government firm should not be included in the estimation of National Income, as it is a transfer payment. Q.21 Case Based Question Intermediate goods can be used in production, but they can also be consumer goods. How it is classified depends on who buys it. If a consumer buys a bag of sugar to use at home, it is a consumer good. But if a manufacturer purchases sugar to use during the production of another product, it becomes an intermediate good. Capital goods, on the other hand, are assets that are used in the production of consumer goods. That means they are purchased to help in the production process. So, the baker who bakes the bread in the example above will buy an oven to use in the production process. That oven is considered a capital good, which doesn't transform or change shape, unlike the wheat. Economists do not factor intermediate goods when they calculate gross domestic product (GDP). GDP is a measurement of the market value of all final goods and services produced in the economy. The reason why these goods are not part of the calculation is that they would be counted twice. Q1. Bread purchased by a household will be considered as __________ consumption. Ans: Final Q2. The addition of the value of bread purchased by a baker while calculating GDP will lead to ___________. Ans: Double-counting Q3. Oven is a _______ good when it is purchased by a baker and a _________ good when it is purchased by a household. Ans: Final Producer/ capital good and Final Consumer good

Q4. The good will be considered as intermediate or final, depends on the _______ of the product. Ans: end-use Q.22 Case study: Is GDP a good indicator of Economic Growth? How uniform is the distribution of GDP? It still seems that majority of the people are poor and only some have benefited. GDP and Welfare Can the GDP of a country be taken as an index of the welfare of the people of that country? If a person has more income, he or she can buy more goods and services and his or her material well- being improves. So it may seem reasonable to treat his or her income level as his or her level of well-being. GDP is the sum total of value of goods and services created within the geographical boundary of a country in a particular year. It gets distributed among the people as incomes (except for retained earnings). So we may be tempted to treat higher level of GDP of a country as an index of greater well-being of the people of that country (to account for price changes, we may take the value of real GDP instead of nominal GDP). Answer the following questions based on the given case: 1. It is_______ (reasonable/unreasonable) to treat a person's income level as his or her level of well-being? 2. In the given picture, all of the people have benefited. True or false 3. In the given picture, the distribution of GDP is______________ (uniform/unequal). 4. GDP is a true indicator of welfare.(choose the correct alternative) i. Agree ii. Partially Agree iii. Disagree iv. Partially disagree Answer: 1. Unreasonable 2. False 3. Unequal 4. Disagree

Q.23 Answer the following questions on basis of below given diagram of circular flow of income in two sectors of the economy. a) From the diagram, name the items which are called money flow. b) From the diagram, name the items which are called real flow. c) How money flow different to real flow d) What do you mean by transfer payments? Give example. Answer a) Money flow items – factor payment inform of rent, wages, interest, profit from firms and expenditure from household. b) Real flow items – FOP from houseld and goods/services from firms. c) money flow involves the exchange of money between two sectors where as real flow is only exchange of goods and services between two sectors without involvement of money. d) Transfer payments are unilateral payment which are non-earned incomes. Ex – oldage pension, scholarship. Q.24 In a single day Arman, a saloon owner, collects Rs 3000 from haircuts, over this day, his equipment depreciates in value by Rs 200. Of the remaining Rs 2800, Arman pays sales tax worth Rs 300, takes home Rs 1500 and retains Rs 1000 for improvement and buying of new equipment. He further pays Rs 200 as income tax from his income. Based on this information, complete Arnav‘s contribution to the following measures of income a. How much is the contribution of Arnav to GDP at MP? b. How much is the contribution NNP at market price c. How much does Arnav contributes to domestic income of the country. d. The income earned by Arnav from his saloon as per income method of national income accounting will be considered as ____________. Answer a) GDPmp = 3000

b) NNPmp = 2800 c) NDPfc = 2500 d) Mixed Income Q.25 Case Study Based Question The Indian Economy saw its worst contraction in decades, with GDP shrinking by a record 23.9% in the April to June Quarter in comparison to the same period last year. The contraction reflects the severe impact of the Covid 19 lockdown, which halted most economic activities as well as the slowdown trend of the economy even pre Covid 19. The Indian economy is in a deeply vicious cycle, where demand is contracting so heavily, while the capacity to neutralise this contraction has also contracted equally because of the tax revenue contraction. Agriculture was the only sector which recorded modest growth of 3.4% in year on year terms and all other sectors saw contraction. Q.1 The impact of Covid 19 on India‘s GDP ______________ (increasing or decreasing). Q.2 _________ is crucial for revenue collection. (Choose the correct alternative) a) National Income b) Gross Value Added c) Net Value Added d) Nominal GDP Q.3 Who are considered as the owner of Factors of Production? a) Firms b) Households c) Government d) Rest of the world Q.4 Net Indirect Taxes are calculated as________________ ANSWERS 1) GROWING 2) (d) Nominal GDP 3) b) Households 4) IDT - Subsidies Q.26 First Advance Estimates of National Income, 2019-20 The National Statistical Office (NSO), Ministry of Statistics and Programme Implementation has released the First Advance Estimates of National Income at both Constant (2011-12) and Current Prices, for the financial year 2019-20 along with the corresponding estimates of expenditure components of the Gross Domestic Product (GDP). Gross Domestic Product Real GDP or GDP at Constant Prices (2011-12) in the year 2019-20 is likely to attain a level of ₹147.79 lakh crore, as against the Provisional Estimate of GDP for the year 2018-19 of ₹140.78 lakh crore, released on 31st May 2019. The growth in real GDP during 2019-20 is estimated at 5.0 per cent as compared to the growth rate of 6.8 per cent in 2018-19.

Gross Value Added (GVA) at Basic Prices Real GVA at Basic Prices is estimated to increase from ₹129.07 lakh crore in 2018-19 to ₹135.40 lakh crore in 2019-20. Estimated growth of real GVA in 2019-20 is 4.9 per cent as against 6.6 per cent in 2018-19. Gross Domestic Product GDP at Current Prices in the year 2019-20 is likely to attain a level of ₹204.42 lakh crore, as against ₹190.10 lakh crore in 2018-19 showing a growth rate of 7.5 per cent. National Income The nominal Net National Income (NNI), also known as National Income (at Current Prices) is likely to be ₹181.10 lakh crore during 2019-20, as against ₹168.37 lakh crore for the year 2018- 19. In terms of growth rates, the National Income registered a growth rate of 7.6 per cent in 2019-20 as against the previous year‘s growth rate of 11.3 per cent. Based on the data released by the ―Press Information Bureau, Government of India ,Ministry of Statistics & Programme Implementation‖, answer the questions given below: i) A growing country is one with: a) Rising GNP at constant prices b) Constant GNP at constant prices c) Rising GNP at current prices d) None of these Ans. a) Rising GNP at constant prices ii) Which of the following statements is correct: a) GDP is derived by subtracting Taxes on Products net of Subsidies on Products to Gross Value Added at Basic Prices. b) GDP is derived by adding Taxes on Products and Subsidies on Products to Gross Value Added at Basic Prices. c) GDP is derived by adding Subsidies net of Taxes on Products to Net Value Added at Basic Prices. d) GDP is derived by adding Taxes on Products net of Subsidies on Products to GVA at Basic Prices. Ans. d) GDP is derived by adding Taxes on Products net of Subsidies on Products to GVA at Basic Prices. iii) National product at current prices is higher than national product at constant prices during a period of: a) Rising prices. b) Falling prices. c) Constant prices. d) Both a and b.

Ans. a) Rising prices. iv) Value added means value of: a) Output at market prices. b) Output less depreciation. c) Output less intermediate cost. d) Output plus intermediate cost. Ans. c) Output less intermediate cost. Case Studies (4 1. Q.27 Read, the following hypothetical Case Study, carefully and answer the below question on the 1 fo basis of the same. q MSME contributes about 30-35% to our GDP, while engaging more than 6 lakh micro, small and medium units, out of which 49% are located in rural and 51% in urban areas. All India Manufacturers Organization survey (AIMO survey June 2020) shows since March, 2020, around 35% of MSMEs and 43% of self-employed have been under closure resulting into 12 crore unemployed masses. The worst- hit have been consumer goods, readymade garments and logistic companies. The Service sector enterprises are still in a better position, trading atleast, slowly though. Similarly, the Online/Internet businesses and start-ups share approx.. 950 billion US$ in our GDP. The nationwide lockdown has tremendously affected the operations of the E-commerce industry especially at a time when there is a huge demand for home delivery of goods. Q.1. High unemployment in industrial sector will result in high ________ (poverty/richness) in agriculture sector or tertiary sector. Q.2. Will Make in India Scheme help in growth of the MSME sector? Give reason. Q.3. The contribution from secondary sector will _________( increase / decrease) in this year‘s GDP. Q.4. Why E-commerce Industry is affected in spite of a huge demand of home delivery goods? Give reason. Ans: 1. Poverty 2. Yes, as Make in India scheme will provide subsidies to necessary MSME industries and thus will help them to grow. 3. Decrease 4. Because the production in secondary sector is reducing and service sector trading is slow. Q28 Case study: It has been observed by leading economists of the world that standard measures of national income such as gross domestic product (GDP) fail to capture important environmental and social factors. This can result in misleading measurements of national well-being. UK has offered variety of methods that can be used to correct GDP measures or to provide alternatives. It believes that estimates of natural capital depreciation measure the depletion of natural resources such as oil, timber, minerals, and agricultural soils, in monetary units. Figures for these losses should be subtracted from the standard measures of national income and investment. The results for many developing countries will indicate a substantial impact of natural resource depletion and environmental degradation. For developed countries, expenditures on pollution control and cleanup, as well as the cumulative impacts of long-lived pollutants, are significant factors. It is

also possible to estimate the value of environmental services such as water purification, nutrient recycling, flood control, and provision of wildlife habitat. Systematic calculation of such factors can give a measure of social progress that often differs significantly from GDP. Social as well as environmental conditions affect calculations of national income. Natural resources may be sold below their true costs, leading to a net loss for the country despite an apparent trade surplus. Q.1 How can trade surplus in current account balance of a country impact national income of that country? (1) Ans: Trade surplus in current account implies net inflow of forex, so it will have a positive impact on National Income and National income may rise. (1) Q.2 Give meaning of externalities. Ans: Externalities refer to benefits (positive externalities) or harms (negative externalities) which are caused by one unit to another with no payment received for the benefit and no payment made for the harm. Q.3 Identify two negative externalities from above case study. (1) Ans: Two negative externalities mentioned in above case study are: Pollution, Soil depletion due to excessive use of chemicals to increase agricultural production. Q.4 Why do you think that green GDP is a better measure of a country‘s growth? Take reference from above case study to explain your answer. (1) ANS: Green GDP can be a better measure for the welfare of the people of a country. Green GDP = GDP – Net fall in stock of Natural capital It helps to achieve a sustainable use of natural environment and equitable distribution of National income. Q.29 New Delhi: Asserting that the ― worst is behind us‖ veteran banker Deepak Parekh onThursday said that India‘s GDP growth will be in positive territory in the fourth quarter as demand has started picking up in all sectors . citing an example he said during the virtual conference organized by Canada – India- based council. He said toll collections are back to 88 percent of pre COVID 19 level , e way bills are increasing and electricity consumption has stabilized. India is domestic based economy and demand is set to rise in coming future. Source : 9 october 2020, The Economic Times On the basis of above case let answer the following questions (4 M) E. ___________the growth of GDP, greater is the flow of goods and services. F. There is no difference between GDP at market price and GDP at factor cost in two sector economy. True/ false. Justify

G. With every increase in the level of GDP , social welfare definitely increases in the economy.(True/ False) justify H. Real GDP is considered as an index of_________________. Answers: A. Higher B. True: difference is of net indirect taxes NIT= IT- subsidies C. False: if increase in GDP is associated with higher level of income inequality, social welfare may not increase. D. Welfare of people. Q.30 One particularly misleading number-play was on the forefront — that US GDP contracted 32 per cent, while India only contracted 23.9 per cent in the June quarter. The Indian economy contracted nearly a quarter in April-June, shrinking more than expected. After recording the first GDP contraction in more than 40 years, India's Q1 performance is now eliciting different responses from different quarters. Based on the above case study answer the following questions Q.1 what is the reason for such contraction in GDP ? (1) Ans : the reason for contraction in India‘s GDP is COVID - 19 , which has drastically affected the employment situation and overall production capacity. Q.2 How the Real GDP and nominal GDP differ from each other. (1) Ans : Real GDP is calculated on the basis of base year price whereas Nominal GDP is calculated on the basis of current years price Real GDP is far more better option to compare the GDP Q.3 which situation it creates in the economy ? (1) Ans. This has created a situation of depression in the economy or rather Deflation. Q.4 what are the possible measures that can be taken by government to tackle such situation (1) Ans : To overcome this situation govt. Can take following steps : 1. Reduction in taxes 2. Increase in govt. Expenditure

Q.31 Read, the following Case Study, carefully & answer the below question on the basis of the same. In a peculiar trend, the inequality gap in India is expected to narrow after the Covid-19 pandemic even as per capita income (PCI) is projected to decline by 5.4% during the ongoing fiscal to Rs 1.43 lakh, according to a State Bank of India (SBI) Research report released on Tuesday. This was attributed to the likely higher PCI decline of around 10-12% in rich states like Maharashtra and Tamil Nadu compared to a less than 8% decline for relatively poorer states like Uttar Pradesh and Bihar, the report said. According to the report, it takes an economy longer to recover in PCI terms to normal levels after a crisis as compared to an average of four years to reach pre-crisis gross domestic product (GDP) levels. Questions: Q.1. The GDP of India _____________ (rise/fall) due to Covid 19. Q.2 The contribution from different sector will (increase/ decrease) in this year. Q.3. Per Capita Income calculated as = Q.4. The PCI of an economy is an important metric as it indicates the standard of living and the _____________ status of a country. Answers: 1. Fall 2. Tertiary 3. Total National Income Total Population 4. development Q.32 (4 Read, the following hypothetical Case Study, carefully and answer the below question on the basis 1 of the same. fo q While the COVID-19 pandemic is constantly growing and showing little signs of containment as of 15 April 2020, its adverse impact on economic growth of the country will probably be very serious. The UN warned that the coronavirus pandemic is expected to have a significant adverse impact on global economy, and most significantly, GDP growth of India for the present economy is projected to decline to 4.8 per cent (United Nation 2020). Economic Survey 2019–2020 had provided advance estimates for growth in real GDP during 2019– 2020 at 5.0 per cent, as compared to the growth rate of 6.8 per cent in 2018–2019. The nominal GDP is estimated at `204,400 billion in 2019–2020 with a growth of 7.5 per cent over the provisional estimates of GDP ( `190,100 billion) for 2018–2019. (Economic Survey, 2020) On 28 February 2020, the National Statistical Office announced revised estimates of GDP growth, from 8 per cent to 7.1 per cent in the first quarter, from 7 per cent to 6.2 per cent in the second quarter and from 6.6 per cent to 5.6 per cent in the third quarter. Goldman Sachs estimated the growth rate of GDP at 1.6 per cent, declining by 400 basis points because of 21-day lockdown (Goldman Sachs, 2020).

Mr. Yashwant Sinha, former Finance Minister of India, estimated the cost of 21-day countrywide lockdown at 1 percentage point of GDP. The global recession and uncertainties of future might make a 2 percentage point decline in growth rate (for 2020–2021) possible. Q.1. The above case study on impact of Covid 19 on India‘s GDP indicates ____________ GDP. a. Increase in GDP at higher rate than previous year b. Fall in GDP than previous year c Increase in GDP at slower rate than past years d Fall in GDP in coming few years Q.2. What is the difference between Real GDP and Nominal GDP Q.3. As per the United Nations estimate, what would be the estimated value of Nominal GDP in the year 2020-21. Q.4. Do you think increase in Real GDP ensures increase in economic welfare? .Ans 1 c. Increase in GDP at slower rate than previous years Ans2 The gross value of goods and services produced within domestic territory when calculated at constant (base year ) prices is known as Real GDP. Whereas the gross value of goods and services produced within domestic territory when calculated at current prices is known as Nominal GDP Ans3: 214,211 billion Ans 4: No, Real GDP does not ensure increase in welfare as it does not consider many factors like per capita GDP , externatilities, non monetaryexchages etc. Q.33 Read the following excerpt carefully and answer the questions [4] that follow on your understanding of the same. National income of India constitutes total amount of income earned by the whole nationals of our country and originated both within and outside its territory during a particular year. The National Income Committee in its first report wrote, ―A national income estimate measures the volume of commodities and services turned out during a given period, without duplication.‖ The estimates of national income depict a clear picture about the standard of living of the community. The national income statistics diagnose the economic ills of the country and at the same time suggest remedies. The rate of savings and investment in an economy also depends on the national income of the country. Moreover, the national income measures the flow of all commodities and

services produced in an economy. Thus, the national income is not a stock but a flow. It measures the total productive power of the community during given period. Further, the National Income Committee has rightly observed, ―National income statistics enable an overall view to be taken of the whole economy and of the relative positions and inter-relations among its various parts‖. Thus, the computation of national income and its analysis has been considered an important exercise on economic literature. Q1. Which territory has been referred to in the first passage of the expert? Q2. Differentiate between a Stock and a Flow. Q3. The returns for the productive power is -------------- income. (transfer/factor) Q4. In a closed economy, what are savings and investments referred to? Answers: A1. Domestic Territory A2. Stock is measured at a particular point of time and flow is measured for a particular period of time. A3. Factor A4. Leakages and Injections Q.34 Read, the following hypothetical Case Study, carefully and answer the below question on the basis of the same. Using the World Bank's headline projections about India, of 3.2% contraction in 2020-21 and 3.1% growth in 2021-22, it becomes easier for us to calculate the projected growth for different sectors of the economy. This comes to a 7.2% contraction in 2020-21 and 1.4% growth in 2021-22. Since the big jolt given by pandemic Covid-19 in March 2020, India has been facing a huge decline in government revenues and growth of the income. Businesses across the world namely hospitality, entertainment, aviation etc., have seen a major negative impact. Various sports events such as IPL and Olympics have been postponed. Schools and colleges have been closed. The virus has also disrupted the functioning of various online giants. There is a big shift in the world economic market and the share market has witnessed crashes day by day. Factories, Restaurants, Pubs, Markets, Flights, Super Markets, Malls, Universities and Colleges etc. have been shut down for 2 quarters. In this backdrop, how fragile is India's unorganized sector, has been sufficiently proven when on 27 May 2020, lakhs of daily wagers, migrant workers and people of small means, on beingdeprived of food and shelter, were constrained to move back to their native places. Undoubtedly, this has been the biggest recession after Second World war.

The COVID-19 pandemic pushed our economy into a Great Lockdown, which helped contain the virus and save lives, but triggered the worst recession since the Great Depression of 1930. It faces an uphill task in its battle to recover from the COVID-19 setback. Q.1. The impact of Covid 19 on India‘s GDP is ____________( growing or declining). Q.2. The impact on Factor income from abroad will __________________(increase or decrease) Q.3. Will there be increase in taxes by the Government? Give reason. Q.4. The government should provide more subsidies or reduce subsidies? Give reason. Ans: 1. Growing 2. Decrease 3. Yes as there is huge decline in government revenues this year. 4. Students can give both answer but reason should be valid. (a) Should provide subsidies as the economy is in recession. (b) Should not provide subsidies as the government has huge decline in government revenues. Q.35 Read, the following hypothetical Case Study, carefully and answer the below question on the basis of the same. Using the World Bank's headline projections about India, of 3.2% contraction in 2020-21 and 3.1% growth in 2021-22, it becomes easier for us to calculate the projected growth for different sectors of the economy. This comes to a 7.2% contraction in 2020-21 and 1.4% growth in 2021-22. Since the big jolt given by pandemic Covid-19 in March 2020, India has been facing a huge decline in government revenues and growth of the income. Businesses across the world namely hospitality, entertainment, aviation etc., have seen a major negative impact. Various sports events such as IPL and Olympics have been postponed. Schools and colleges have been closed. The virus has also disrupted the functioning of various online giants. There is a big shift in the world economic market and the share market has witnessed crashes day by day. Factories, Restaurants, Pubs, Markets, Flights, Super Markets, Malls, Universities and Colleges etc. have been shut down for 2 quarters. In this backdrop, how fragile is India's unorganized sector, has been sufficiently proven when on 27 May 2020, lakhs of daily wagers, migrant workers and people of small means, on being deprived of food and shelter, were constrained to move back to their native places. Undoubtedly, this has been the biggest recession after Second World war. The COVID-19 pandemic pushed our economy into a Great Lockdown, which helped contain the virus and save lives, but triggered the worst recession since the Great Depression of 1930. It faces an uphill task in its battle to recover from the COVID-19 setback. Q.1. The impact of Covid 19 on India‘s GDP is ____________( growing or declining). Q.2. The impact on Factor income from abroad will __________________(increase or decrease) Q.3. Will there be increase in taxes by the Government? Give reason. Q.4. The government should provide more subsidies or reduce subsidies? Give Reason. Ans: 1. Growing 2. Decrease 3. Yes as there is huge decline in government revenues this year. 4. Students can give both answer but reason should be valid. (a) Should provide subsidies as the economy is in recession. (b) Should not provide subsidies as the government has huge decline in government revenues.

Q.36 Read, the following hypothetical Case Study, carefully and answer the below question on the basis of the same. MSME contributes about 30-35% to our GDP, while engaging more than 6 lakh micro, small and medium units, out of which 49% are located in rural and 51% in urban areas. All India Manufacturers Organization survey (AIMO survey June 2020) shows since March, 2020, around 35% of MSMEs and 43% of self-employed have been under closure resulting into 12 crore unemployed masses. The worst-hit have been consumer goods, readymade garments and logistic companies. The Service sector enterprises are still in a better position, trading atleast, slowly though. Similarly, the Online/Internet businesses and start-ups share approx.. 950 billion US$ in our GDP. The nationwide lockdown has tremendously affected the operations of the E-commerce industry especially at a time when there is a huge demand for home delivery of goods. Q.1. High unemployment in industrial sector will result in high ________ (poverty/richness) in agriculture sector or tertiary sector. Q.2. Will Make in India Scheme help in growth of the MSME sector? Give reason. Q.3. The contribution from secondary sector will _________( increase / decrease) in this year‘s GDP. Q.4. Why E-commerce Industry is affected in spite of a huge demand of home delivery goods? Give reason. Ans: 1. Poverty 2. Yes, as Make in India scheme will provide subsidies to necessary MSME industries and thus will help them to grow. 3. Decrease 4. Because the production in secondary sector is reducing and service sector trading is slow. Q.37 Prices are rising at the fastest rate in 7 years while job loss and pay cuts due to the long lockdown have reduced people‘s means. In the first quarter of 2020-21, GDP shrank by 24% and the estimates of the second quarter are also not encouraging. In October 2020, CPI inflation touched 7.6%, a record for Modi years. Inflation in the country had been largely under control since 2014. Retail price inflation had breached the 6% mark only once-in July 2016- between September2014 and November 2019. In May 2019, the inflation rate was 1.9% in rural India and 4.5% in cities. The gap went on reducing and in October 2020 inflation in rural India was at 7.7% as against 7.4% in cities. It seems the steep price rise in villages is a major cause of the high inflation now. A. __________ measures the average change in the price level of all goods and services that constitute GDP. It is also known as _________. B. During Covid 19 pandemic, India is experiencing ________ and yet prices are rising. C. How GDP expressed in terms of the current market value of the quantities can be converted in physical terms? D. The main reason of high inflation in India in 2020 is due to price rise in _______. Answers: A. GDP Deflator, Consumer Price Index. B. Recession/ Stagflation. C. Real GDP=Nominal GDP*100/Price Index of current year D. Villages.

Q.38 CASE STUDIES BASED QUESTIONS AND ANSWERS 1. Unanticipated localised lockdowns across the country is likely to raise uncertainty amongst the manufacturers and various other stakeholders in the supply chain who had planned to resume their operations. As per government data, retail inflation had increased to 6.09 per cent in June, mainly on account of higher prices of food items. Suppose you are a member of the high-powered committee constituted by the Reserve Bank of India (RBI). You have suggested that as the supervisor of commercial banks, ____________ (restriction/release) of the money supply be ensured, by the Reserve Bank of India (RBI). (Choose the correct alternative) Ans: Release 2. Turning to growth in India, the implied real GDP growth of 4.7 per cent for 2019-20 in the second advance estimates of the National Statistics Office, released in February 2020, within the annual estimate of 5 per cent for the year as a whole is now at risk from the pandemic‘s impact on the economy. As regards the outlook for 2020-21, apart from the continuing resilience of agriculture and allied activities, most other sectors of the economy will be adversely impacted by the pandemic, depending upon, I repeat, its intensity, spread and duration. If COVID-19 is prolonged and supply chain disruptions

get accentuated, the global slowdown could deepen, with adverse implications for India. The slump in international crude prices could, however, provide some relief in the form of terms of trade gains. Downside risks to growth arise from the spread of COVID-19 and prolonged lockdowns. Upside growth impulses are expected to emanate from monetary, fiscal and other policy measures and the early containment of COVID-19. The Economic Times; March 27th, 2020 I. To obviate the adverse impact of pandemic on the economy, RBI is likely to decrease ____. a) Cash Reserve Ratio b) Rationing of Credit c) Both (a) and (b) d) None of these Ans: a II. The continuing resilience of agriculture and allied activities will lead to_______. a. Rise in aggregate supply b. Fall in aggregate supply c. No change in aggregate supply d. Fall in general price level Ans: b III. The__________ is vested with the responsibility of conducting monetary policy. a) Reserve Bank of India b) Finance Commission c) NITI Aayog d) Both (a) and (c) Ans: a IV. _____________(Aggregate demand/Aggregate supply) refers to total ex-ante expenditure in an economy during an accounting year. Ans: Aggregate demand Q.39 TANWA is a project initiated to train women in latest agricultural techniques. It induces women to actively participate in raising agricultural productivity and family income. Women are forming Farm Women‘s Groups, which function like SHG. Many other Farm Women‘s Groups are creating savings in their group by functioning like mini banks. With the accumulated savings, they promote small-scale household activities: (like mushroom cultivation, soap manufactures, doll making or other income- generating activities) 1. Self help groups are voluntary organisation. State true or false. Ans: True 2. The main aim of self help group is to :

a. Eradicate poverty b. Maximise profit c. Set up industries d. Create competition for cooperative banks Ans: a 3. SHG is a type of_____________ Source of rural credit. a. Institutional b. Non-institutional c. Both of the above d. None Ans: a Q.40 Every society has to answer three questions:  What goods and services should be produced in the country?  How should the goods and services be produced? Should producers use more human labour or more capital (machines) for producing things?  How should the goods and services be distributed among people? In a market economy, also called capitalism, only those consumer goods will be produced that are in demand, i.e., goods that can be sold profitably either in the domestic or in the foreign markets. If cars are in demand, cars will be produced and if bicycles are in demand, bicycles will be produced. If labour is cheaper than capital, more labour- intensive methods of production will be used and vice-versa. In a socialist society the government decides what goods are to be produced in accordance with the needs of society. The government decides how goods are to be produced and how they should be distributed. In principle, distribution under socialism is supposed to be based on what people need and not on what they can afford to purchase. In Cuba and China, for example, most of the economic activities are governed by the socialistic principles. In a mixed economy, the market will provide whatever goods and services it can produce well, and the government will provide essential goods and services which the market fails to do. An economy is a system in which individuals earn a source of livelihood by performing economic activities. State true or false. I. India has adopted ____ Economy system for its development. a. Capitalist b. Socialistic c. Mixed d. Free Ans: b

II. In which type of economy there is no economic problem found? a. Mixed b. Capitalist c. Socialistic d. None Ans: d III. In a market economy the economic problems are solved by government intervention. State true or false Ans: false IV. An economy is a system in which individuals earn a source of livelihood by performing economic activities. State true or false. Ans:True Q.50 The government of India, as well as the state governments, initiate several schemes that assist to diminish the plight of the farmers. Thus, it focuses on every segment of the agricultural scheme. Such as, each subdivision from raw material procurement to the transport and marketing of the produce, subsidies are available. Further, Haryana, the highest agricultural producer in the country is adopting modernized machinery to encourage agricultural advancements. Thus, few schemes like RKVY, crop diversification, State plan, NFSM, are ISOPOM aiding the cause. Central Govt and the State Govt are trying to make farmers more self-reliant and empowered, with the aim of which the government launched the PM Kisan Tractor Yojana 2020. Under the PM Kisan Tractor Yojana 2020, the government provides subsidy on the purchase of new tractors from 20 to 50 percent in a state and 50 percent elsewhere. This subsidy is up to 50 percent of central and state government inclusive. Q 1 purchase of a tractor by a farmer will be considered as a)final good b) capital good c)intermediate good d)both (a) and (b) Q2 which one of the following will be referred to as intermediate goods : a) fertilizers used by farmers b) wheat used by flour mill c) wheat with the retailer d) all the above Q3Which of the following will not be included while calculating national income a) wheat grown by farmer but used entirely for families consumption

b) wheat grown by farmers to sell in market c) growing vegetables in the kitchen garden of the house d) family members working free on farm owned by family. Q4. If factor cost is greater than market price , then it means that: a) indirect taxes>subsidies b) Indirect taxes=subsidies c) Indirect taxes <subsidies d) Indirect taxes >=subsides ANSWERS Q1 d) both(a) and (b) Q2 d) all the above Q3c) growing vegetables in the kitchen garden Q4c) indirect taxes<subsidies Q.51 It is sometimes advocated that only the production of those goods should be included in the estimation of national income which flow from producers to the consumers through the market. Consider an economy where farming is the only production activity. Also, assume that all the farmers in the economy produce only for self consumption: they do not sell their produce in the market, because production is barely enough for consumption by the farming families themselves. Thus final goods (rice or wheat) are being consumed without any market transaction. If only such goods which are routed through the market are to be recorded, such an economy (where production is only for self consumption) would be a zero production economy. It sounds ridiculous. Can an economy survive without production? Let it be clearly understood that the imputed value of production for self consumption is always included in the estimation of national income. 1.Corresponding to production for self consumption, there should be generation of................ (factor income / transfer income) in the economy. (Choose the correct alternative) 2. Imputed value of production for self consumption is taken into account for estimation of national income because it's a .............. . a. Transfer Income b. Earned Income c. Unearned income d. Non marketable transaction 3. Is any economy in the world a Zero production economy? (True /False)

4. It is difficult to estimate market value of ............... for self consumption. a. Production of consumer goods b. Production of services c. Production of capital goods d. Production of green vegetables Answers: 1. (Factor Income) 2.( Earned Income) 3.(False) 4. ( Production of services) Q.52 Income method accounts for only factor incomes corresponding to which there is flow of goods and services in the economy. It does not account for such incomes corresponding to which there is no flow of goods and services or corresponding to which there is no value addition in the economy. All taxes by the government are compulsory transfer payments. These are not to be included in the estimation of N.I. Answer the following questions: 1. Transfer payments are_____________________(included/not included) while estimating N.I. 2. Factor of production are contributing to change in GDP.(True/False) 3. Money received as gift from Grand Mother is ___________________ (Transfer income/Factor income). 4. Taxes are _________________________(compulsory/voluntary) payments. 5. Define factor income Answer: It is the income received by owners of factor of production in form of rent, wages, interest and profit for the services rendered in production process. Q.53 There are only two sectors in the economy: Households and Firms. It means the government and foreign sector. Household sector supplies factor services only to firms and the firms hire factor services only from households. Firms produce goods and services and sell their entire output to the households. Households receive factor income for their services and spend the entire amount on consumption of goods and services. There are no savings in the economy, i.e. neither the households save from their incomes, nor the firms save from their profits.

Q1. Name the two dominant sectors of the economy? Ans. The two main sectors of Indian economy are: 1) Household sector 2) Firms Q2. What are factor payments? Ans. Factor payments are the income received for supplying the factors of production. Q3. What is the importance of consumption expenditure? Ans. It helps in creating demand in the economy. Q4. What would happen if householders will be removed from the two sector model of economy? Ans. If householders will remove, then the purchasing power in an economy will also decline. Q.54 Read the following contend and answer the following questions: Gujarat‘s Kevadia, where Sardar Patel‘s Statue of Unity has been drawing more tourists than the Statue of Liberty in the US,has emerged as a family holiday destination of international standard with a children nutrition park, Arogya Van and camping and river rafting facilities, top officials said . Described as a ‗ must – visit ‗ place by Prime minister Narendra Modi , the town nestled amid Satpura and Vindhyachal ranges along the bank of the Narmada river, has a number of small and big tourist spots. Development in the town generated direct employment for 3000 tribal boys and girls, and indirect employment to 10,000 more . It has also opened up new avenues of micro-entrepreneurship for women. For nature lover, there is Sardar Patel Zoological Parks and Geodesic Aviary Dome, which is home to more than 1100 birds and animals and 5lakh plants. Q1. How does tourism industry contribute to economy of the India? Ans. 1.Increases employment and income 2. Promotes local entrepreneurship Q2. When foreign tourists buy from local markets, it increases our _________( exports, factor income from abroad). Ans. Exports Q3. If a tourist get attached to Indian cultural beauty and decides to live here, what conditions are required for him to become Normal Resident of India? Ans. 1. He should reside in India for more than one year. 2. His centre of economic interest should lie here.

Q.55 Read, the following hypothetical Case Study, carefully and answer the following questions on the base of the same. It is possible that an income, which is a part of domestic income of India, is not included in the national income. Similarly, an income which is a part of national income, may not be included in domestic income it happened because national income includes the income of normal residents only (irrespective of their place of earning) where as domestic income includes the income of both residents and non- residents (but, within the domestic Territory of the country) let us clear this point with the help of an example: Included in national income but not in domestic income Salary received by an Indian employee working in the Japanese Embassy in India. It will be included in the national income as he is a resident of India. of the factor However, such an income will not be included in India‘s domestic income as the Japanese Embassy is not a part of the domestic territory of India. i)Profits earned by a branch of foreign bank in India will be included in National Income or not? ii)Profits of a branch of State bank of India in England will be included in Domestic Income or not? iii)Rent received by a Indian from his building in London will be included in Domestic Income or not? iv)Purchase of goods by a foreign tourist will be included in Domestic Income or not? Answers Q1. a. Q2.i) It will not be included in National Income as it is a part of the factor income payed abroad. Foreign bank is not a resident of India. ii) It will not be included in Domestic Income as SBI branch in England is not a part of the domestic territory of India. iii) No, it will not be in included in Domestic Income as the rent is earned outside the domestic territory of India. Q.56 A government budget is announced and government has allocated over ₹ 15,000 crores for the construction of hospitals and related infrastructure and announced different initiative to make health sector better. Government has also announced to open some community yoga centres for which ₹ 500 crore allotted separately and planning to hire some qualified yoga instructors on a salary of approx. ₹ 30,000 pm. For the same purpose, government allotted 50 crores for awareness campaigning programmes through various media channels to make yoga more popular among public. In the same budget, government also announces to give scholarship to the students who would like to pursue yoga as their profession in future. Q: 1 Is government expenditure of ₹ 15000 crores part of the national income. If yes, then mention the category under which it will be recorded. Q:2 Define Real flow and Money flow. Hiring yoga instructors @ ₹ 30,000 represents which flow? Q:3 Is scholarship given to the students included in national income? Give reason in support of your answer.

Ans1 Yes. Government expenditure of ₹15000 crores for construction of hospitals will be included in national income. It will be recorded under Gross Domestic Capital formation as Gross public investment. Ans.2In a simple economy, the flow of factor services from households to firms and corresponding flow of goods and services from firms to households is known as real flow. Money flow of income represents a monetary payment from firms to households for their factor services and in return monetary payments from households to firms against their goods and services. Hiring yoga instructor represents real flow while salary represents money flow Ans.3 Expenditure on scholarship is a transfer payment for the government as it has no contribution in the current flow of goods and service. Therefore it will not include in the national income Q.57 \" The turnaound in India's manufacturing sector,which rebounded from a 39.3% contraction in GVA in the April- June quarter,has left economists scrambling to reconcile conflicting data from the government as they try to come to term with the latest GDP estimate that suggests the economy fared better than expected\"..The Hindu 28th November 2020 a. Identify the method used in calculating the GDP in the above case study. (1) b. How we reconcile three methods of calculating National Income.(1) c. State four precautions to take while calculating GDP with the above method (method used in the case study) . Q.58 Read the following Case Study and answer the question number 2 – 5 on the basis of the same. GDP and Welfare Can the GDP of a country be taken as an index of the welfare of the people of that country? If a person has more income, he or she can buy more goods and services and his or her material well-being improves. So it may seem reasonable to treat his or her income level as his or her level of well-being GDP is the sum total of value of goods and services created within the geographical boundary of a country in a particular year. It gets distributed among the people as incomes(except for retained earnings). So we may tempted to treat higher level of GDP of a country as an index of greater well-being of the people of that country(to account for price changes, we may take the value of real GDP instead of nominal GDP). Q.7 Do you think that GDP is a true indicator of welfare? Yes or No? Q.8 _____ is an example of ―externality‖ which reduces welfare of the people. Q.9 GDP Deflator (a) Evaluates inflation by utilizing present production basket (b) Shows real GDP growth on the basis of current production

(c) The GDP deflator is in real terms (d) None of the above Q.10 The formula for calculating GDP deflator is ______. Q.59 India GDP Q2 Data: India‘s Gross Domestic Product (GDP) for the July-September quarter (Q2) contracted by 7.5 per cent following an unprecedented decline of 23.9 per cent in the April- June quarter, as per provisional estimates released by the Ministry of Statistics and Programme Implementation (MoSPI) on Friday. The GDP had expanded by 5.2 per cent in the corresponding quarter of 2019-20. With this latest development, the Indian economy has entered into a technical recession for the first time in history. In economics, when the GDP growth rate is negative for two consecutive quarters or more, it is termed a recession. The GDP during the April-June quarter (Q1) had contracted by 23.9 per cent, the worst contraction in the history of the Indian economy, owing to a strict nationwide lockdown due to the novel coronavirus (COVID-19) during the bulk of the quarter. However, the nation has gradually moved out of the lockdown phase and the government has gradually eased several restrictions, with the boiler rooms of the economy sputtering back to life. On Thursday, Reserve Bank of India (RBI) Governor Shaktikanta Das said the economy had exhibited a stronger-than-expected pick-up in momentum of recovery but cautioned about the ―sustainability of demand‖. Interestingly, the RBI had ‗nowcast‘ that GDP for the July-September quarter was set for a contraction of 8.6 per cent. The RBI, however, said the economy would break out of contraction of the six months gone by and return to positive growth in the October-December quarter of 2020-21. Q1. When is an economy said to be in recession?(1) Ans . When the gdp growth rate is negative for two consecutive quarters Q2. When is the economy expected to have positive growth rate?(1) Ans. Oct - Dec quarter of 2020-21 Q3. Why is 'sustainability of demand' a concern?(2) Ans. Demand has increased after lockdown and the govt is concerned that the demand might go down after sometime as the income levels have reduced (as we can see through gdp contraction due to covid 19) Q.60 Covid 19 has adversely affected the Indian economy. Initially there was drastic downfall in the demand of non vegetarian food items. Restaurants, hotels , transportation , theatres , cab companies etc were suffering a lot due to extreme decrease in their demand. Many people lost their jobs leading to shortage of cash in hand. To control the situation during such pandemic our Government distributed food among the poor people. However there was high demand of sanitizer, mask etc and e-commerce has been doing great during this time.

a. Due the covid 19, the GDP of india has ____ (decreased/increased) b. There was _______ in Net Exports of India during this pandemic. (Fall/Rise) c. Explain the changing trend in contribution of various sectors in national income during Covid 19. d. How did it affect the Government final consumption expenditure? Ans. a. Decreased b. Fall c. In order to maintain social distancing and lockdown there was fall in demand for services like cab, restaurants, hotels, theatres , hence their contribution to National Income reduced where as e- commerce was doing great as most of people were doing shopping sitting at home hence its contribution increased. Also,masks , sanitizers , thermometers etc has contributed much more than usual. d. Government final consumption expenditure increased as government had to distribute food as well as cash amongst the poor people for their survival. Q.61 In 2015, in the wake of a comprehensive review of its approach to GDP measurement, India opted to make major changes to its compilation of national accounts and bring the whole process into conformity with the United Nations System of National Accounts (SNA) of 2008. As per the SNA, gross value added, is defined as the value of output minus the value of intermediate consumption and is a measure of the contribution to GDP made by an individual producer, industry or sector. At its simplest it gives the rupee value of goods and services produced in the economy after deducting the cost of inputs and raw materials used. GVA can be described as the main entry on the income side of the nation‘s accounting balance sheet, and from an economics perspective represents the supply side. While India had been measuring GVA earlier, it had done so using ‗factor cost‘ and GDP at ‗factor cost‘ was the main parameter for measuring the country‘s overall economic output till the new methodology was adopted. In the new series, in which the base year was shifted to 2011-12 from the earlier 2004-05, GVA at basic prices became the primary measure of output across the economy‘s various sectors and when added to net taxes on products amounts to the GDP. 7 June 2020: The Hindu. 1. The difference between value of output and value added is________(depreciation/intermediate cost/ 2. Product method of calculating national income is also known as________(value added method/expenditure method) 3. Net value added at factor cost= Net value of output at market price-_________( NFIA/NIT) 4. National income is represented by_______(NNPfc/GDPfc) Q.62 India‘s economy contracted by 7.5% (It was 23.9% decline of April-June) for the second successive quarter but the pace of decline narrowed sharply in three month period ending. India‘s

economy still remains in the league of large economies which have contracted the most in the second quarter. China is only large economy that posted growth in September quarter. ―Expectedly, the worst for the economy is over. That said, the economy is still not out of the woods and the improvement in activity from here onward would likely be laboured,‖ said KunalKundu, India economist at French Investment Bank SocieteGenerale. In the second quarter the farm sector held strong while manufacturing returned to the positive zone. However, the crucial services segment (hotels, restaurants cinemas etc.) fell low. The services sector which accounts nearly 60% of GDP by 11.4% narrower than the 20.6% in the previous quarter This sector has borne the brunt of the pandemic as hotels, restaurants, cinemas has been shuttered for the longest time and people stayed away due to the fear of infections. Private consumption, a key driver of the economy, contracted 11.3% from an expansion of 6.4% in the year ago period while government consumption showed sharp decline in growth 22.2% in the June quarter. Investment growth continued to contract with grows capital formation. Some economists said they expect growth to improve in the remaining two quarters of 2020-21 as economic activity improves across sectors. 1. Define the following terms (a) Gross Domestic Product (b) Capital formation (c) Services (d) Consumption (e) Economic activity (f) Final goods 2. Explain the circular flow of income of income in two sector economy with emphasis on leakages and injections. 3. What is economic growth rate? How is it affected by shrinking of economy? 4. What do you mean by private final consumption expenditure? How can it play important role in reviving economy? National income is the total market value of production in a country‘s economy during a year. It can be measured alternatively and equivalently in three ways: The value of expenditures The value of inputs used in production The sum of value added at each level of production Q.63 That the first two measures are identical can be seen by considering that any good—say, a loaf of bread—can be equivalently valued as either the price that is paid for it in the market by the final consumer or as the distributed factor payments—to labour (wages) and to capital (rent, interest, and profit)—used in its production. Since national output is the sum of all production, the total value will be the same whether added up by final expenditure or by the value of inputs (including profit) used in their production. The equivalence of the last measure can be seen by noting that the value of every final good is simply the sum of the value added at each stage of production. Again, consider a loaf of bread: Its value is the sum of the value of labour at each successive stage of production and other ingredients added by the farmer (wheat production), the miller (grinding to flour), the baker (flour plus other ingredients), and the grocer (distribution services).

Q 1. ‗A final consumer is always a household.‘ True/ false. Give reason. Q2. Factor payments are made by ________ sector to ________ sector. Q3. Wheat used by miller is _______ good. Q4. Value added method solves the problem of ________ of wants. 1. False. A producer is also a final consumer consuming capital goods 2. Producer, household 3. Intermediate 4. Double coincidence Q.64 It is sometimes advocated that only the production of those goods should be included in the estimation of national income which flow from producers to the consumers through the market. Consider an economy where farming is the only production activity. Also, assume that all the farmers in the economy produce only for self consumption: they do not sell their produce in the market, because production is barely enough for consumption by the farming families themselves. Thus final goods (rice or wheat) are being consumed without any market transaction. If only such goods which are routed through the market are to be recorded, such an economy (where production is only for self consumption) would be a zero production economy. It sounds ridiculous. Can an economy survive without production? Let it be clearly understood that the imputed value of production for self consumption is always included in the estimation of national income. 1.Corresponding to production for self consumption, there should be generation of................ (factor income / transfer income) in the economy. (Choose the correct alternative) 2. Imputed value of production for self consumption is taken into account for estimation of national income because it's a .............. . a. Transfer Income b. Earned Income c. Unearned income d. Non marketable transaction 3. Is any economy in the world a Zero production economy? (True /False)

4. It is difficult to estimate market value of ............... for self consumption. a. Production of consumer goods b. Production of services c. Production of capital goods d. Production of green vegetables Answers: 1. (Factor Income) 2.( Earned Income) 3.(False) 4. ( Production of services) Q.65 The government of India, as well as the state governments, initiate several schemes that assist to diminish the plight of the farmers. Thus, it focuses on every segment of the agricultural scheme. Such as, each subdivision from raw material procurement to the transport and marketing of the produce, subsidies are available. Further, Haryana, the highest agricultural producer in the country is adopting modernized machinery to encourage agricultural advancements. Thus, few schemes like RKVY, crop diversification, State plan, NFSM, are ISOPOM aiding the cause. Central Govt and the State Govt are trying to make farmers more self-reliant and empowered, with the aim of which the government launched the PM Kisan Tractor Yojana 2020. Under the PM Kisan Tractor Yojana 2020, the government provides subsidy on the purchase of new tractors from 20 to 50 percent in a state and 50 percent elsewhere. This subsidy is up to 50 percent of central and state government inclusive. Q 1 purchase of a tractor by a farmer will be considered as a)final good b) capital good c)intermediate good d)both (a) and (b) Q2 which one of the following will be referred to as intermediate goods : a) fertilizers used by farmers b) wheat used by flour mill c) wheat with the retailer d) all the above Q3Which of the following will not be included while calculating national income a) wheat grown by farmer but used entirely for families consumption

b) wheat grown by farmers to sell in market c) growing vegetables in the kitchen garden of the house d) family members working free on farm owned by family. Q4. If factor cost is greater than market price , then it means that: a) indirect taxes>subsidies b) Indirect taxes=subsidies c) Indirect taxes <subsidies d) Indirect taxes >=subsides ANSWERS Q1 d) both(a) and (b) Q2 d) all the above Q3c) growing vegetables in the kitchen garden Q4c) indirect taxes<subsidies Q.66 Read the following hypothetical case study carefully and answer the following questions on the base of the same: GDP Is Not a Measure of Human Well-Being Economic growth has raised living standards around the world. However, modern economies have lost sight of the fact that the standard metric of economic growth, gross domestic product (GDP), merely measures the size of a nation‘s economy and doesn‘t reflect a nation‘s welfare. Yet policymakers and economists often treat GDP, or GDP per capita in some cases, as an all- encompassing unit to signify a nation‘s development, combining its economic prosperity and societal well-being. Environmental degradation is a significant externality that the measure of GDP has failed to reflect. The production of more goods adds to an economy‘s GDP irrespective of the environmental damage suffered because of it. So, according to GDP, a country like India is considered to be on the growth path, even though Delhi‘s winters are increasingly filled with smog. Q.1 Is GDP a true indicator of economic welfare? State one reason with regard to the example cited above. (1) Ans. No, GDP is not always a true indicator of economic welfare and one of the reasons for this is that GDP does not account externalities, positive or negative. Q.2 What do you mean by externality? (1) Ans. Externality refers to the benefits or harms which a firm or an individual cause to other or to the nature but for which they are not paid or penalized.

Q.3 In the example of Delhi cited above which externality is talked about? (1) Ans. In the example of Delhi cited the concept of negative externality is being discussed. Q.4 How does this affect the welfare of the people of Delhi and GDP of an economy? (1) Ans. Delhi has been contributing largely to the national GDP by producing a lot of goods and services but this has also lead to increased pollution in the city and with the smog lot of people face health issues and hence welfare of the people is adversely affected. Q.67 Read the following case study paragraph carefully and answer the questions on the basis of the same. GDP is the most closely monitored aggregate of the economy. Covid-19 Pendemic causes economic crisis rather than a health crisis. India‘s GDP showing alarming downturn of 23.9 % which is all time high in past 40 years( As per report of 30th. Sept, 2020). India is a country which has to incur huge maintenance expenditure being second largest populative country of the world. Covid 19 has added to it further. After reform india‘s service sector developed a lot. Service sector accounts 54.40% of GVA. Lockdown and restrictions have reduced the movement across the country and to other countries as well. Hence causes loss of income and fall in production to . Due to fall in production in domestic economy there is increase in imports by domestic economy. 1. Name the sector of economy that still has shown the minimum sign of revival after pendemic. (a)Agriculture (b) Travel and tourism (c) Banking (d) Industries Answer (b) Travel and Tourism 2. High maintenance expenditure cause reduction in ________(Infrastructural investment/defence ) expenditure of a country which further reduces development procedure. Answer Infrastructural investment expenditure. 3. Economic crisis here refers to (a)Shortage of funds with migrant labourers to go to their native towns. (b) Less capability to work (c) Education sector fails to develop skills (d) Fall in productivity, Production and standard of living. Answer (d) 4. What will be the impact of rise in imports from domestic country on the national income?

(a) Rise (b) Fall (c) Remain same (d)May rise or Fall Q.68 A resident is said to be a person or an institution who ordinarily resides in a country and whose center of economic interest lies in that country. He is called a normal resident since he normally lives in the country of his economic interest. During this COVID-19, Many people returned to their home country, which has been shown by the following article: Migrants – particularly in lower-paid jobs – may be both more affected by and vulnerable to the spread of COVID-19 in countries already impacted and those countries where the pandemic is spreading, but migrants also play an important role in the response to COVID-19 by working in critical sectors. As of 3 November 2020, emigrants from the 20 countries with the highest number of COVID-19 cases accounted for nearly 28 percent of the total international migrant stock and they had sent an estimated 37 percent of all remittances globally to their countries of origin in 2019 (GMDAC analysis based on UN DESA, 2019; World Bank 2020a; WHO, 2020) Immigrants accounted for at least 4.5 percent of the population in 12 of the 20 countries with the highest number of COVID-19 cases, and this share is more than 10 percent in 8 of these countries (GMDAC analysis based on UN DESA, 2019; WHO, 2020). Compared to the global share of international migrants making up 3.5 percent of the total population, international migrants are overrepresented in these countries. Increasing border restrictions also have an impact on the mobility of migrants and the role of humanitarian organizations. Between 11 March 2020, when the WHO declared COVID-19 a pandemic, and 26 October 2020, the total number of movement restrictions implemented around the world has increased to more than 96,000. At the same time, 167 countries, territories, or areas have issued 681 exceptions to these restrictions, thus enabling mobility. Another issue may be returning migrant workers, who have been working overseas, over half of whom work in the Gulf. It is unclear if, or when, migrants will be able to return to work, with the World Bank estimating that remittances from this group could fall by about 23%. However, what is striking has been India‘s support for this group - the Vande Bharat Mission has deployed flights and naval ships to help return migrant workers, especially vulnerable groups - in marked contrast to the lack of preparation and care for internal migrants. One factor for this may be the volume of remittances these migrant workers bring to the Indian economy, but it overlooks the contribution of internal remittances, on which there is far less robust data. On the above concept, answer the following cases should be included or not included in estimation of National Income with appropriate reason: 1. Profits earned by a resident of India from his company in Denmark. 2. Profits earned by a resident of Canada from his company in India . 3. Salaries to Indian residents working in the Australian embassy in India. 4. Profits earned by A branch of State Bank of India in New Zealand. Answer: 1. No, it will not be included as the company is located outside the domestic territory of India. 2. Yes, it will be included as profits are earned within the domestic territory of India. 3. No, it will not be included as Australian embassy is not part of India's domestic territory. 4. No, it will not be included as State Bank of India is located outside the domestic territory of India. Read the following statements - Assertion (A) and Reason (R). Choose one of the correct alternatives given below:

Q.69 Since in a monetary economy, all payments are made in money the real flow turns itself into money flow of income. When firms get factor services from households, they make monetary payments against these services to the households. These payments are made in the form of wages to workers, rent to landowners, interest to capitalists, and profit to entrepreneurs. These monetary payments are factor incomes for households. Households spend this income on the purchase of goods and services from the firms for their consumption. As the households spend all their income on consumption of goods and services, total money receipt of the firm is the same as the total income of the households. In other words, the money delivered to households by firms as factor payments come back to them when households spend their income on consumptions. Q1. National income is a _______ concept. (Stock / Flow) Q2. Choose the other name of real flow. A) Nominal flow B) Money flow C) Cash flow D) Physical flow Q3. Which two sectors of the economy are being talked in the above paragraphs? Ans.Households and Firms Q4. Choose the odd one out. A) Entrepreneur B) Capital C) Profit D) Labour Q.70 Double counting in accounting is an error whereby a transaction is counted more than once for whatever reason. But in social accounting it also refers to a conceptual problem in social accounting practice, when the attempt is made to estimate the new value added by Gross Output, or the value of total investments. Read the following paragraph and answer the following questions a) Explain the method of Double counting with an example. ( 2) b) How can the problem of double counting be avoided and explain it briefly ? ( 2) Ans a) Double counting method explanation with example table b). To avoid the problem if double counting, we can use the two methods 1. Value added method - According to this method, sum total of the value added by each producing unit should be taken in the national income.

2. Final Output method - According to this method, value of only final goods should be added to determine the national income. Assertion( A): Houses are self occupied so they are treated income generating assets and are counted in calculation of national income Reason( R): Rental income is assessed on the basis of rent prevailing in the market. Both A and R are true and R is the correct explanation of A. Answer: The rental income assessed on the basis of rent prevailing in the market is known as imputed rent on owner occupied houses and is treated as a part of factor income generated during the year and is included in calculation of national income. So both the statements are true and reason is correct explanation of assertion Therefore the correct option is A). Q.71 India‘s quarterly GDP was estimated to a decline of over nine percent between April and June 2020. This was a decrease from a five percent growth in the beginning of 2020. The country went into lockdown on March 25, 2020, the largest in the world, restricting 1.3 billion people. This was extended until May 3, 2020. India‘s government estimated its financial, real estate and professional services sector (Health and Hospital, Pharmaceutical, Biotechnology, Financial sector, Insurance Sector, IT sector – Online / Digital Education, Digital Marketing, AI, Robotics etc.) to be hardest hit during the period of the lockdown. Q-1 Once lockdown is lifted, how will productivity rise? Q-2 Which sectors of the Indian Economy will grow faster? Q-3 Do you think this pandemic is affecting our circular flow of income. Give reason. Q-4 The impact of covid-19 will give rise to more of ..........(unemployment / employment) and........ (inflation / deflation). Ans 1: Once lockdown is lifted, firms may not be able to operate immediately or atleast till thebeginning 2021, at the earlier full capacity due to a variety of reason. This will cause elimination or retrenchment of a number of employees. Thus, the operation will operate at a lower capacity with less no of employees resulting in a productivity increase. Ans 2: Financial, real estate and professional services sector (Health and Hospital, Pharmaceutical, Biotechnology, Financial sector, Insurance Sector, IT sector – Online / Digital Education, Digital Marketing, AI, Robotics etc.) Ans 3:3Yes, reason is upto the person And 4: unemployment & inflation Q.72 Payment of interest by Bank to its depositors or payment of interest by a Farm to the households or a bank is included in the national income. The borrowed money is used for carrying out the production of goods and services. However, the payment of interest on loan taken by a consumer or payment of interest on a loan taken by an employee from its employer is

not to be included in the national income as such loan is taken out to meet the consumption expenditure. There is no contribution to production of goods and services. (I) Which type of interest payment is included in national income? (II) Payment of interest on car loan taken by an individual is included in national income (true or false) (III) Payment of interest on loan taken for the construction of a new house is to be included in national income. Why? (IV) Payment of interest on loan by bank to household will be included because it is____________(factor income/ transfer income) Q.73 Read, the following hypothetical Case Study, carefully and answer the below (4) 1 mark question on the basis of the same. for each question. Nominal and real GDP Money (or nominal) GDP is GDP measured in terms of the prices operating in the year in which output is produced. It is sometimes referred to as GDP at current prices and is a measure that has not been adjusted for inflation. Money GDP may give a misleading impression of how well a country is performing. This is because the value of money GDP may rise not because more goods and services are being produced but merely because prices have risen. For example if 100 billion products are produced at an average price of ₹5, GDP will be ₹500 billion. If in the next year the same output of 100 billion products is produced but the average price rise to ₹6, money GDP will rise to ₹600 billion. So to get a truer picture of what is happening to output, economists convert money into real GDP. They do this by measuring GDP at constant prices, meaning the prices operating in a selected year. By doing this they remove the distorting effect of inflation. For example, in 2016 a country's GDP is ₹800 billion and the price index is 100. Then in 2017, money GDP is ₹900 billion and the price index is 120. Real GDP = Nominal GDP X price index in base year price index in current year Real GDP = 900 X 100 = ₹ 750 Billion 120 The price index used to convert money into real GDP is called the GDP deflator, which measures the prices of products produced rather than consumed in a country. So it includes the prices of capital goods as well as consumer products and includes the price of exports but excludes the price of imports. Q1: Define Real GDP. Q2: Why does GDP deflator excludes imports? Q3: In 2019 a country‘s GDP is ₹1000. In 2020 nominal GDP rises to ₹1092 and the price index increase by 4%. Calculate real GDP. Q4: In 2019 a country‘s GDP is ₹1000. In 2020 nominal GDP rises to ₹1092 and the price index increase by 4%. Calculate the percentage increase in real GDP. Answer1: Real GDP is GDP measured in terms of the prices operating in a

selected year (base year). Answer2: GDP deflator measures the prices of products produced rather than consumed in a country. So it includes the price of exports but excludes the price of imports. Answer3: 1092 X 100 Real GDP = 104 ������������������������ ������������������ = ₹ ������������������ Answer4: Real GDP in 2019 = 1000 Real GDP in 2020 = 1050 50 X 100 % Increase in real GDP = 100 % ������������������������������������������������ ������������ ������������������������ ������������������ = ������������% Q.74 The outbreak of COVID-19 brought social and economic life to a standstill. In this study the focus is on assessing the impact on affected sectors, such as aviation, tourism, retail, capital markets, MSMEs, and oil. International and internal mobility is restricted, and the revenues generated by travel and tourism, which contributes 9.2% of the GDP, will take a major toll on the GDP growth rate. Aviation revenues will come down by USD 1.56 billion. Oil has plummeted to 18-year low of $ 22 per barrel in March, and Foreign Portfolio Investors (FPIs) have withdrawn huge amounts from India, about USD 571.4 million. While lower oil prices will shrink the current account deficit, reverse capital flows will expand it. Rupee is continuously depreciating. MSMEs will undergo a severe cash crunch. The crisis witnessed a horrifying mass exodus of such floating population of migrants on foot, amidst countrywide lockdown. Their worries primarily were loss of job, daily ration, and absence of a social security net. India must rethink on her development paradigm and make it more inclusive. COVID 19 has also provided some unique opportunities to India. There is an opportunity to participate in global supply chains, multinationals are losing trust in China. To ‗Make in India‘, some reforms are needed, labour reforms being one of them. Q1. Explain the impact of COVID -19 pandemic on GDP and Migrants Labour. Answer: India had also been witnessing a pre-pandemic slowdown. Even before the pandemic, since FY 2018–19, India's growth was falling, 8% in Q4 FY18 to 4.5% in Q2 FY20. In January 2020 itself, well before India's lockdown or reactions to the pandemic, the IMF reduced India's GDP estimates for 2019 and also reduced the 2020 GDP forecast. The 2016 Bank note demonetization and GST enactment in 2017 led to severe back to back disruptions in the economy.

Due to the lockdown, daily-wage workers (the urban poor and migrant laborers) were left with no work. At the same time, the lockdown restrictions put a stop on the movement of buses and trains. Large numbers of migrant workers ended up walking back to their villages. On 20 June 2020 the government launched the GaribKalyanRojgarAbhiyaan for the welfare of migrants. In July, Mint (The Financial Newspaper Reported) that companies were having difficulties in bringing back the workforce. Even after incentives, many laborers are reluctant to travel back to urban areas. Q2. Explain implication on capital Market, Global oil Market and its impact on India. Answer: Weak or reduced oil prices have a major positive impact on the Indian economy. India being an importer of crude oil, so higher oil prices imply, more payment needs to be made in foreign currency. And oil prices have a major say in the financial markets of our country. A weak oil price usually signals strength in the performance of the stock market. And a strong oil price has a negative impact on the performance of the stock market. And similarly, if we were to take the example of oil-exporting nations, strong oil prices have a very positive effect on their incomes, balance of payments, and their financial markets. Assertion and Reason based questions 1. Assertion(A): Domestic income of a country can be more than its National income. Reason(R): It is possible when factor income paid abroad is more than factor income received from abroad a) Both (A) and (R) are true and (R)is the correct explanation of (A) b) Both (A) and (R) are true and (R) is not the correct explanation of (A) c) Both (A) and (R) are false d) (A) is wrong (R) is true Ans: Option A is correct 2. Assertion(A): Growing vegetables in a kitchen garden of the house is included in national income Reason (R): It is difficult to estimate value of production as it is non market transaction a) Both (A) and (R) are true and (R)is the correct explanation of (A) b) Both (A) and (R) are true and (R) is not the correct explanation of (A) c) (A) is false and (R) is correct explanation of (A) d) (A) is true (R) is not correct explanation of (A) 1. (A) The sum of final expenditures in the economy must be equal to the incomes received by all the factors of production taken together (R) The revenues earned by all the firms put together must be distributed among the factors of production as salaries, wages, profits, interest earnings and rents

2. (A ) Final expenditure is that part of expenditure which is undertaken not for intermediate purposes. ( R ) Intermediate goods do not fall under the category of final expenditure. 3. ( A )Change in inventories may be planned or unplanned. ( R ) In case of an unexpected fall in sales, the firm will have unsold stock of goods which it had not anticipated. 4. ( A ) If we include depreciation in value added, then the measure of value added that we obtain is called Gross Value Added. (R ) Depriciation or Consumption of fixed Capital is inclusive in Net Value Added Q.75 Let us suppose that there are only two kinds of producers in the economy. They are the wheat producers (or the farmers) and the bread makers (the bakers). The wheat producers grow wheat and they do not need any input other than human labour. They sell a part of the wheat to the bakers. The bakers do not need any other raw materials besides wheat to produce bread. Let us suppose that in a year the total value of wheat that the farmers have produced is Rs 100. Out of this they have sold Rs 50 worth of wheat to the bakers. The bakers have used this amount of wheat completely during the year and have produced Rs 200 worth of bread. If we follow the simple way of aggregating the values of production of the sectors, we would add Rs 200 (value of production of the bakers) to Rs 100 (value of production of farmers). 1. What is the value of total production in the economy?( Rs 100/Rs 200/Rs 300) 2. Intermediate goods are used (in /outside) the production process 3. The problem of double counting can be avoided by atking the value of --------- (intermediate consumption/final goods) Answers: 1. Rs 300 2. Outside 3. Final goods

Q.76 Gross Domestic Product measures the aggregate production of final goods and services taking place within the domestic economy during a year. But the whole of it may not accrue to the citizens of the country. For example, a citizen of India working in Saudi Arabia may be earning her wage and it will be included in the Saudi Arabian GDP. But legally speaking, she is an Indian. Is there a way to take into account the earnings made by Indians abroad or by the factors of production owned by Indians? When we try to do this, in order to maintain symmetry, we must deduct the earnings of the foreigners who are working within our domestic economy, or the payments to the factors of production owned by the foreigners. For example, the profits earned by the Korean-owned Hyundai car factory will have to be subtracted from the GDP of India. The macroeconomic variable which takes into account such additions and subtractions is known as Gross National Product (GNP). Q 1. What is the difference between domestic product and national product? Q 2. When is GNP < GDP? Q 3. Will the income earned by an American Company in India included in National Income of America? Answer: 1. NFIA 2. When NFIA value is negative 3. Yes. As it is the resident company of America Ans: Option C is correct Q.77 Read the following statement and answer Questions 7-10 on the basis of the same: One‘s ability to buy commodities comes from the income one earns as labourer (earning wages), or as entrepreneur (earning profits), or as landlord (earning rents), or as owner of capital (earning interests). In short, the incomes that people earn as owners of factors of production are used by them to meet their demand for goods and services. Q7. The statement above is based on the notion‖ one‘s expenditure is other man‘s income. (true /false) Q8. This exchange is depicted as ----------(real flow/money flow) Q9. Using up of incomes earned in consumption of goods is called …….( income distribution/income disposal) Q10. All the factors of production are owned by ------ household/firms) Ans: 7. True 8. money flow 9. income disposal 10. households


Like this book? You can publish your book online for free in a few minutes!
Create your own flipbook